Why is the relativistic Lagrangian for a free particle proportional to 1/\gamma?

In summary: I'm not sure about the details of this argument, but essentially, the only way for the Lagrangian to satisfy both conditions is for it to be proportional to 1/\gamma. This is because the Lagrangian must be independent of position, which means it can only be a function of velocity and mass. To be a Lorentz scalar, \gamma \mathcal L must be invariant under Lorentz transformations. The only available Lorentz invariant function of the 4-velocity is c^2 = v_\mu v^\mu. Therefore, the Lagrangian for a free particle must be \mathcal L = -mc^2 / \gamma. The reason for (2) and (3) is not
  • #1
AxiomOfChoice
533
1
In J.D. Jackson's Classical Electrodynamics, an argument is made in support of the assertion that the relativistic Lagrangian [itex]\mathcal L[/itex] for a free particle has to be proportional to [itex]1/\gamma[/itex]. The argument goes something like this:
  1. [itex]\mathcal L[/itex] must be independent of position and can therefore only be a function of velocity and mass.
  2. [itex]\gamma \mathcal L[/itex] must be a Lorentz scalar.
  3. The only available Lorentz invariant function of the 4-velocity is [itex]c^2 = v_\mu v^\mu[/itex].
From this, it is (according to Jackson) "obvious" that the relativistic Lagrangian for the free particle has to be
[tex]
\mathcal L = -mc^2 / \gamma.
[/tex]
I guess I can see why this should be the case, given that the Euler-Lagrange equations need to be satisfied and that the Lagrangian needs to have the appropriate units. What I *don't* get is where (2) and (3) come from. Can someone please explain?
 
Last edited:
Physics news on Phys.org
  • #2
(When I initially made this post, my question was incomplete. I've updated it. Sorry!)
 
  • #3
Can you give a page number? I can't seem to find what you're referring to.

#3 seems obvious to me, but not #2. Re #3, what other scalar could you make out of the 4-velocity besides c? (Of course, you could make c5, etc.) Re #2, I don't know, but I'm guessing this is because the action has to be a scalar...?
 
  • #4
Sure. It's pgs. 580-81.

A question about (3)...am I to infer that the only Lorentz invariant function of any 4-vector is its scalar product?
 
  • #5
AxiomOfChoice said:
Sure. It's pgs. 580-81.

Hmm...on pp. 580-581, I have the tail end of section 12.2, "On the question of obtaining the magnetic field..." This is in the 2nd ed. of Jackson. I don't see the material you're referring to. Do you have a later edition?

AxiomOfChoice said:
A question about (3)...am I to infer that the only Lorentz invariant function of any 4-vector is its scalar product?

I think so, except that you can obviously take the norm of a 4-vector and push it through any function you like that accepts a scalar input and is itself Lorentz-invariant. E.g., you can multiply the norm of the v 4-vector by the mass and get the norm of the momentum, or take the norm of the v 4-vector to the 5th power.
 
  • #6
Yes, I think we've got different editions; I'm pretty sure I'm using the 3rd Ed. The discussion I'm talking about shows up at the start of Chapter 12, in any event.
 
  • #7
Okay, on p. 573, the second page of section 12.1 in my 2nd edition of the book, I have an argument that [itex]A=\int_{\tau_1}^{\tau_2} \gamma L d \tau [/itex], and since A and [itex]\tau[/itex] are supposed to be invariant, [itex]\gamma L [/itex] has to be invariant as well. This seems reasonably sensible to me, although I wouldn't have had the confidence to state the same argument in the same somewhat breezy form without having thought long and hard about all the details that are not explicitly given. E.g., it seems plausible to me that we need to require Lorentz invariance for A, but it's not obvious that this is really true. If someone told me that A could be non-invariant, but all the predictions of the theory about experimental observables would still be 100% invariant, I wouldn't have had a snappy comeback to prove they were wrong.
 
  • #8
bcrowell said:
Okay, on p. 573, the second page of section 12.1 in my 2nd edition of the book, I have an argument that [itex]A=\int_{\tau_1}^{\tau_2} \gamma L d \tau [/itex], and since A and [itex]\tau[/itex] are supposed to be invariant, [itex]\gamma L [/itex] has to be invariant as well. This seems reasonably sensible to me, although I wouldn't have had the confidence to state the same argument in the same somewhat breezy form without having thought long and hard about all the details that are not explicitly given. E.g., it seems plausible to me that we need to require Lorentz invariance for A, but it's not obvious that this is really true. If someone told me that A could be non-invariant, but all the predictions of the theory about experimental observables would still be 100% invariant, I wouldn't have had a snappy comeback to prove they were wrong.

Ok. But I can understand an attempt to make the Lagrangian Lorentz invariant. (Is "covariant" another word for "Lorentz invariant"?) What I can't quite understand is this statement: "since A and [itex]\tau[/itex] are supposed to be invariant, [itex]\gamma L [/itex] has to be invariant as well." WHY is this true? I have a pretty rigorous training in higher mathematics, and it's trained me to need (and demand) a REASON for this! Maybe the integral can do something strange to [itex]\mathcal L[/itex] that makes it unnecessary for [itex]\gamma \mathcal L[/itex] to be Lorentz invariant...I certainly can't think of any reason why this can't happen!
 
  • #9
AxiomOfChoice said:
Is "covariant" another word for "Lorentz invariant"?

More or less. The term is used kind of loosely by physicists, with several different, but related, definitions. "General covariance" refers to the property of GR that its predictions are invariant under any smooth change of coordinates. "Covariant" can also be used as the opposite of "contravariant," to describe tensors with upper versus low indices. In the present context, however, it basically means the same thing as "Lorentz invariant."

AxiomOfChoice said:
What I can't quite understand is this statement: "since A and [itex]\tau[/itex] are supposed to be invariant, [itex]\gamma L [/itex] has to be invariant as well." WHY is this true? I have a pretty rigorous training in higher mathematics, and it's trained me to need (and demand) a REASON for this! Maybe the integral can do something strange to [itex]\mathcal L[/itex] that makes it unnecessary for [itex]\gamma \mathcal L[/itex] to be Lorentz invariant...I certainly can't think of any reason why this can't happen!

Yeah, I agree that Jackson is leaving a lot to the reader's imagination. I think it might be more transparent if you rewrite the equation for the action in differential form, as [itex]dA/d\tau=\gamma L[/itex]. The left-hand side is the quotient of two infinitesimally small numbers. (I hope you're not allergic to infinitesimals. They can be just as rigorous as limits, as shown by Robinson in the 60's.) Now [itex]d\tau[/itex] is Lorentz-invariant, and say we want dA to be Lorentz-invariant as well. Then dividing these two Lorentz-invariant quantities gives another Lorentz scalar. Therefore [itex]\gamma L[/itex] has to be a scalar as well. This is a very common mode of reasoning in relativity. You start with things that you know behave as well-defined tensors, and combine them to get new tensors. E.g., [itex]m dv/d\tau[/itex] is constructed out of a scalar, a 4-vector, and a scalar, so it produces a valid 4-vector (the momentum 4-vector).

The main thing that sticks out to me as maybe needing more justification in my own reasoning above is that it's not necessarily obvious that dA has to be Lorentz-invariant, or even that A does. The only experimental observables are essentially incidence relations between world-lines, i.e., do they cross or not. It's conceivable that A and/or dA could be non-Lorentz-invariant, and yet you'd get observables that would be Lorentz-invariant. But I think the general philosophy is that all the machinery of tensors and least-action are designed so that you never, ever write anything down on the paper that isn't *manifestly* a valid relativistic equation.
 

1. What is the relativistic Lagrangian?

The relativistic Lagrangian is a mathematical function that describes the dynamics of a system in terms of its position and velocity, taking into account special relativity. It is a key concept in the field of theoretical physics and is used to derive the equations of motion for a system.

2. How does the relativistic Lagrangian differ from the classical Lagrangian?

The relativistic Lagrangian takes into account the effects of special relativity, such as time dilation and length contraction, which are not present in the classical Lagrangian. This makes it a more accurate and comprehensive tool for describing the dynamics of systems moving at high speeds.

3. What is the role of the relativistic Lagrangian in physics?

The relativistic Lagrangian is a fundamental concept in physics that is used to describe the motion of particles and systems in terms of their position and velocity. It is an essential tool for understanding and predicting the behavior of objects in the realm of special relativity.

4. How is the relativistic Lagrangian derived?

The relativistic Lagrangian is derived using the principle of least action, which states that the path taken by a system between two points in space and time is the one that minimizes the action integral. This integral takes into account the kinetic and potential energies of the system, as well as the effects of special relativity.

5. Can the relativistic Lagrangian be applied to all systems?

Yes, the relativistic Lagrangian can be applied to all systems, regardless of their size or complexity. It is a universal tool that is used to describe the dynamics of particles and systems in terms of their position and velocity, taking into account the principles of special relativity.

Similar threads

  • Special and General Relativity
Replies
3
Views
526
  • Special and General Relativity
Replies
18
Views
2K
  • Special and General Relativity
Replies
1
Views
746
  • Special and General Relativity
Replies
15
Views
1K
  • Special and General Relativity
Replies
7
Views
3K
  • High Energy, Nuclear, Particle Physics
Replies
1
Views
1K
  • Advanced Physics Homework Help
Replies
0
Views
658
  • Special and General Relativity
Replies
9
Views
2K
  • Advanced Physics Homework Help
Replies
1
Views
724
  • Quantum Physics
Replies
14
Views
2K
Back
Top